[Rozgrzewka OM][MIX][Nierówności] Nierówności

Zadania z kółek matematycznych lub obozów przygotowujących do OM. Problemy z minionych olimpiad i konkursów matematycznych.
Regulamin forum
Wszystkie tematy znajdujące się w tym dziale powinny być tagowane tj. posiadać przedrostek postaci [Nierówności], [Planimetria], itp.. Temat może posiadać wiele różnych tagów. Nazwa tematu nie może składać się z samych tagów.
Awatar użytkownika
Premislav
Użytkownik
Użytkownik
Posty: 15687
Rejestracja: 17 sie 2012, o 13:12
Płeć: Mężczyzna
Lokalizacja: Warszawa
Podziękował: 196 razy
Pomógł: 5221 razy

Re: [Rozgrzewka OM][MIX][Nierówności] Nierówności

Post autor: Premislav »

Elegancko, można kontynuować.
Tmkk
Użytkownik
Użytkownik
Posty: 1718
Rejestracja: 15 wrz 2010, o 15:36
Płeć: Mężczyzna
Lokalizacja: Ostrołęka
Podziękował: 59 razy
Pomógł: 501 razy

Re: [Rozgrzewka OM][MIX][Nierówności] Nierówności

Post autor: Tmkk »

Pokazać, że z warunku \(\displaystyle{ ab+bc+ca \ge 1}\) wynika nierówność

\(\displaystyle{ \frac{1}{a^2} + \frac{1}{b^2} + \frac{1}{c^2} \ge \frac{\sqrt{3}}{abc}}\)
Awatar użytkownika
Premislav
Użytkownik
Użytkownik
Posty: 15687
Rejestracja: 17 sie 2012, o 13:12
Płeć: Mężczyzna
Lokalizacja: Warszawa
Podziękował: 196 razy
Pomógł: 5221 razy

Re: [Rozgrzewka OM][MIX][Nierówności] Nierówności

Post autor: Premislav »

Ukryta treść:    
Tmkk
Użytkownik
Użytkownik
Posty: 1718
Rejestracja: 15 wrz 2010, o 15:36
Płeć: Mężczyzna
Lokalizacja: Ostrołęka
Podziękował: 59 razy
Pomógł: 501 razy

Re: [Rozgrzewka OM][MIX][Nierówności] Nierówności

Post autor: Tmkk »

Bardzo ładnie.
Awatar użytkownika
Premislav
Użytkownik
Użytkownik
Posty: 15687
Rejestracja: 17 sie 2012, o 13:12
Płeć: Mężczyzna
Lokalizacja: Warszawa
Podziękował: 196 razy
Pomógł: 5221 razy

Re: [Rozgrzewka OM][MIX][Nierówności] Nierówności

Post autor: Premislav »

Niech \(\displaystyle{ a,b,c\in \RR^{+}}\). Proszę wykazać, że
\(\displaystyle{ \frac{a^{3}}{(a+b)^{3}}+\frac{b^{3}}{(b+c)^{3}}+\frac{c^{3}}{(c+a)^{3}}\ge \frac{3}{8}}\).
bosa_Nike
Użytkownik
Użytkownik
Posty: 1666
Rejestracja: 16 cze 2006, o 15:40
Płeć: Kobieta
Podziękował: 71 razy
Pomógł: 447 razy

Re: [Rozgrzewka OM][MIX][Nierówności] Nierówności

Post autor: bosa_Nike »

Ukryta treść:    
Awatar użytkownika
Premislav
Użytkownik
Użytkownik
Posty: 15687
Rejestracja: 17 sie 2012, o 13:12
Płeć: Mężczyzna
Lokalizacja: Warszawa
Podziękował: 196 razy
Pomógł: 5221 razy

Re: [Rozgrzewka OM][MIX][Nierówności] Nierówności

Post autor: Premislav »

Znakomicie (szkoda, że mogę przyznać tylko jeden punkt za ten post), początek (do Höldera włącznie) miałem taki sam, ale to, co robiłem później, jest jak moja twarz – wstyd to pokazywać ludziom, więc gdy mogę tego uniknąć, to tak uczynię. Można kontynuować.
bosa_Nike
Użytkownik
Użytkownik
Posty: 1666
Rejestracja: 16 cze 2006, o 15:40
Płeć: Kobieta
Podziękował: 71 razy
Pomógł: 447 razy

Re: [Rozgrzewka OM][MIX][Nierówności] Nierówności

Post autor: bosa_Nike »

Zadania w stylu poniższego spotkać można w różnych wstępnych zestawach treningowych, może przyda się trochę ogólniejszy wynik. Ostrzegam - powiedzieć, że ten problem nie jest finezyjny, to jak nic nie powiedzieć.

Dane są: całkowite dodatnie liczby \(\displaystyle{ m,n}\) oraz dodatnia rzeczywista liczba \(\displaystyle{ k}\). Znajdź najmniejszą wartość wyrażenia $$ka^m+\frac{1}{(a-b)b^n}$$ dla rzeczywistych \(\displaystyle{ a>b>0}\).
Awatar użytkownika
Premislav
Użytkownik
Użytkownik
Posty: 15687
Rejestracja: 17 sie 2012, o 13:12
Płeć: Mężczyzna
Lokalizacja: Warszawa
Podziękował: 196 razy
Pomógł: 5221 razy

Re: [Rozgrzewka OM][MIX][Nierówności] Nierówności

Post autor: Premislav »

Ukryta treść:    
bosa_Nike
Użytkownik
Użytkownik
Posty: 1666
Rejestracja: 16 cze 2006, o 15:40
Płeć: Kobieta
Podziękował: 71 razy
Pomógł: 447 razy

Re: [Rozgrzewka OM][MIX][Nierówności] Nierówności

Post autor: bosa_Nike »

Św. Wilgefortis pisze:Dla mnie żyleta.
Awatar użytkownika
Premislav
Użytkownik
Użytkownik
Posty: 15687
Rejestracja: 17 sie 2012, o 13:12
Płeć: Mężczyzna
Lokalizacja: Warszawa
Podziękował: 196 razy
Pomógł: 5221 razy

Re: [Rozgrzewka OM][MIX][Nierówności] Nierówności

Post autor: Premislav »

Oby nie było (nie mam cierpliwości, żeby sprawdzać cały temat, w każdym razie przypominam sobie, że zadanie, które sąsiaduje w Powrocie z tym, pojawiło się na forum co najmniej dwa razy, więc może być różnie).

Liczby dodatnie \(\displaystyle{ a,b,c}\) spełniają warunek \(\displaystyle{ a^{2}+b^{2}+c^{2}=1}\). Proszę udowodnić nierówność
\(\displaystyle{ \frac{a}{1-a^{2}}+\frac{b}{1-b^{2}}+\frac{c}{1-c^{2}}\ge\frac{3\sqrt{3}}{2}}\)

Zadanie wydawało mi się kiedyś trudniejsze przez to, że Kourliandtchik używał tu wypukłej powłoki (no ale to tez celem ilustracji zagadnienia, bo temu poświęcony był rozdział), tymczasem istnieje szalenie elementarne rozwiązanie.
blublu
Użytkownik
Użytkownik
Posty: 3
Rejestracja: 13 lis 2020, o 16:27
Płeć: Mężczyzna
wiek: 20
Podziękował: 1 raz
Pomógł: 1 raz

Re: [Rozgrzewka OM][MIX][Nierówności] Nierówności

Post autor: blublu »

Zauważmy, że dla \(\displaystyle{ x \in (0,1)}\):
\(\displaystyle{ \frac{x}{1-x^{2}}\ge\frac{3\sqrt{3}}{2}x^2}\).
Istotnie, przekształcamy równoważnie: \(\displaystyle{ \Leftrightarrow \frac{1}{x-x^{3}}\ge\frac{3\sqrt{3}}{2} \Leftrightarrow x-x^{3} \le \frac{2}{3\sqrt{3}}}\), co jest prawdą.
(maximum funkcji \(\displaystyle{ x-x^3}\) na \(\displaystyle{ (0, 1)}\) wynosi właśnie \(\displaystyle{ \frac{2}{3\sqrt{3}}}\) i jest osiągane w \(\displaystyle{ x= \frac{1}{\sqrt3}}\))
Zatem
\(\displaystyle{ \frac{a}{1-a^{2}}+\frac{b}{1-b^{2}}+\frac{c}{1-c^{2}}\ge\frac{3\sqrt{3}}{2}(a^2+b^2+c^2)=\frac{3\sqrt{3}}{2}}\).

//Ponieważ \(\displaystyle{ a,b,c>0}\) i \(\displaystyle{ a^2+b^2+c^2=1}\), więc \(\displaystyle{ a,b,c \in (0,1)}\).
Ostatnio zmieniony 14 lis 2020, o 21:14 przez Jan Kraszewski, łącznie zmieniany 1 raz.
Powód: Poprawa wiadomości.
Awatar użytkownika
Premislav
Użytkownik
Użytkownik
Posty: 15687
Rejestracja: 17 sie 2012, o 13:12
Płeć: Mężczyzna
Lokalizacja: Warszawa
Podziękował: 196 razy
Pomógł: 5221 razy

Re: [Rozgrzewka OM][MIX][Nierówności] Nierówności

Post autor: Premislav »

To jest wlaśnie to rozwiazanie dużo prostsze i zgrabniejsze niż wypukła powłoka z Powrotu do krainy nierówności. Można kontynuować.
blublu
Użytkownik
Użytkownik
Posty: 3
Rejestracja: 13 lis 2020, o 16:27
Płeć: Mężczyzna
wiek: 20
Podziękował: 1 raz
Pomógł: 1 raz

Re: [Rozgrzewka OM][MIX][Nierówności] Nierówności

Post autor: blublu »

Załóżmy, że \(\displaystyle{ 0< a_{1} \le a_{2} \le a_{3} \le...\le a_{n} }\). Proszę wykazać, że
\(\displaystyle{ \frac{a_{1}}{a_{2}}+\frac{a_{2}}{a_{3}}+...+\frac{a_{n}}{a_{1}} \ge \frac{a_{2}}{a_{1}}+\frac{a_{3}}{a_{2}}+...+\frac{a_{1}}{a_{n}}}\).
Awatar użytkownika
Premislav
Użytkownik
Użytkownik
Posty: 15687
Rejestracja: 17 sie 2012, o 13:12
Płeć: Mężczyzna
Lokalizacja: Warszawa
Podziękował: 196 razy
Pomógł: 5221 razy

Re: [Rozgrzewka OM][MIX][Nierówności] Nierówności

Post autor: Premislav »

Ukryta treść:    
ODPOWIEDZ